Тёмный
No video :(

Weaken | LSAT Logical Reasoning 

LSAT Lab
Подписаться 17 тыс.
Просмотров 36 тыс.
50% 1

Опубликовано:

 

29 авг 2024

Поделиться:

Ссылка:

Скачать:

Готовим ссылку...

Добавить в:

Мой плейлист
Посмотреть позже
Комментарии : 55   
@stephanieblack7196
@stephanieblack7196 4 года назад
these are my favorite lsat videos so far, I like how you organize them with different colors and strong graphics. It looks very professional. I also like your voice, it is easy to listen to
@LSATLab
@LSATLab 4 года назад
Thank you Stephanie!! We'll keep making more!
@lucatripp7129
@lucatripp7129 3 года назад
You all probably dont care but does someone know of a tool to get back into an Instagram account? I was stupid lost the login password. I appreciate any tricks you can offer me!
@lachlanjaziel4147
@lachlanjaziel4147 3 года назад
@Luca Tripp instablaster :)
@lucatripp7129
@lucatripp7129 3 года назад
@Lachlan Jaziel Thanks so much for your reply. I got to the site on google and I'm trying it out now. Looks like it's gonna take a while so I will reply here later when my account password hopefully is recovered.
@jakhongirabdurakhmonov7876
@jakhongirabdurakhmonov7876 3 года назад
Great explanations! I am learning a lot from this content, especially how to think!
@jakhongirabdurakhmonov7876
@jakhongirabdurakhmonov7876 3 года назад
‘’Too weak’’ trap answers - awesome
@alli1docious
@alli1docious 2 года назад
this was a really helpful video. I am having a hard time with the weakening questions.
@C00ligdeSoBased
@C00ligdeSoBased 4 года назад
Omg THANK YOU SO MUCH for these videos 😭
@aa697
@aa697 2 года назад
You already spent over 12 minutes to answer one question out of a possible 26 questions in this section. How can you finish this section in 25 or 30 minutes under this circumstance?
@rellrizzy
@rellrizzy 4 года назад
Cool video. Thanks! Note: $100 X 40 is $4,000, not $40,000
@LSATLab
@LSATLab 4 года назад
Great catch Derrell, thank you!
@tsunamirider2
@tsunamirider2 4 года назад
Dude these videos are clutch thanks for making them
@LSATLab
@LSATLab 4 года назад
So glad you're enjoying them. We have a lot more coming to our RU-vid channel over the next 8 weeks. We have one on Most Supported questions going up today. There are tons more at lsatlab.com available for free if you can't till they go live on RU-vid.
@yassi8690
@yassi8690 Месяц назад
1 am and I got both right 🥲
@LawjqTV
@LawjqTV 5 месяцев назад
This helped alot! Taking mines in August
@myriamcuprill2373
@myriamcuprill2373 3 года назад
All those videos are help me a lot to review Laws Concept...I liked more at CAMPUS.
@gregorymiller2045
@gregorymiller2045 3 года назад
Thank you for the video! May you please further comment on why B is correct. I think it is correct because it is not denying that they can contribute, it's just simply saying there are also other reasons for it. Therefore, it's correct because there still holds the possibility they do contribute to such illness/health issues (trying to put it in my own words). Is this the correct way to think about this answer choice? Thank you again
@therealsherminator
@therealsherminator 3 года назад
Nice Gregory, that's exactly right!
@taylorhawkins397
@taylorhawkins397 3 года назад
Can you explain a little more why E would not be the answer?
@user-yz6ee5qm1s
@user-yz6ee5qm1s Год назад
The confusing part of this question is equating business travellers with those travel on business class. Leisure travellers may travel on business class too.
@mairobertson9349
@mairobertson9349 Год назад
Are we trying to find the gap in between the premises? Or the gap between the conclusion and premises?
@danengel3421
@danengel3421 3 года назад
40 * 100 = 4,000 (not 40,000)
@hamsterpie123
@hamsterpie123 4 года назад
can you elaborate on why D is incorrect in the second example question?
@LSATLab
@LSATLab 4 года назад
Any answer that weakens the argument is incorrect in this EXCEPT question. Answer choice (D) weakens the argument by calling into question how dangerous irradiated foods are by comparing the level of dangerous chemicals in irradiated foods with the level of such chemicals that naturally occur in foods. If the amount of dangerous chemicals in irradiated foods is less, then maybe they're not so bad after all. While it doesn't destroy the argument, it does make the conclusion a little less likely to be true given the evidence.
@DOoD0oHua1
@DOoD0oHua1 2 года назад
I’m still struggling to see why B is the answer for question 2. Isn’t B also weakening the conclusion? Can someone explain to me? Thank you for the video.
@DOoD0oHua1
@DOoD0oHua1 2 года назад
I finally get it. It says “many causes” but not all therefore it is harmful in some ways but not cancer and serious health problems. So it supports the conclusion that there is a good reason to avoid irradiated foods. I would definitely miss this one and pick D. But now I will pay more attention when I do weakening questions.
@LSATLab
@LSATLab 2 года назад
@@DOoD0oHua1, you might be a little confused in how you're talking about (B). The correct answer on weaken EXCEPT doesn't need to strengthen the argument at all. It just has to not-weaken. If there were an answer that said "Mustard is yellow" or "Baseball is a slow sport" those would both be correct answers to question 2. They *don't* weaken the argument. To weaken the argument, we're looking for answers that allow us to disagree with "good reason to avoid irradiated food". We want to eliminate the four answers that help us argue "it's fine to eat irradiated food. you don't need to avoid it" The closest (B) would come to doing that would be saying, "It's fine to eat irradiated food. You don't need to avoid it. Sure it might spawn radiolytic products that can cause cancer, but hey you can get cancer a lot of different ways. Why bother trying to avoid getting it via irradiated food when you'll probably get cancer anyway from some other source!" That's definitely not a common sense logic we use in real life: "You might as well smoke cigarettes. Yes, they can cause cancer, but so can other stuff, so what's the point in trying to avoid cancer?" (D), meanwhile, allows us to say "It's fine to eat irradiated food. You don't need to avoid it. Sure, it has some harmful chemical residues in it, but the amount of harmful chemical residues in irradiated foods is less than the amount that occurs naturally in most kinds of food." This isn't just saying, "You're bound to get harmful chemical residues in your food no matter what, so you might as well eat irradiated foods". It actually is saying that irradiated foods are BETTER than most other kinds of food when it comes to chemical residues.
@ashugokul7925
@ashugokul7925 4 года назад
So the process of weaken questions entails weakening the existing premises or introducing the new premises sometimes which ultimately effects the conclusion? Not to attack the conclusion directly??
@LSATLab
@LSATLab 4 года назад
That's right. We're typically told to undermine the argument or the reasoning, which means we should focus on how the evidence relates to the conclusion. Occasionally we'll be asked to weaken the conclusion. In those cases we can attack the conclusion directly.
@jasonlou2013S
@jasonlou2013S 2 года назад
Thank you so much.
@ToucheTJ
@ToucheTJ 4 года назад
Would "Provides the least support for..." be considered a weakening?
@LSATLab
@LSATLab 4 года назад
Great question! Not exactly. "Provides the least support for..." would be more like Strengthens, EXCEPT. So four answers will provide some support for the argument or statement, the correct answer will not.
@ToucheTJ
@ToucheTJ 4 года назад
@@LSATLab yes i came across a question with that and was wrong because it was supposed to be a weaken queation
@mairobertson9349
@mairobertson9349 Год назад
Hello, so are we attacking the premises or the conclusion for weaken questions? To weaken the conclusion would we say why should… Airlines should focus on business travelers more? Or Airlines shouldn’t focus on leisure travelers
@LSATLab
@LSATLab Год назад
You're just trying to weaken the author's case, in any conceivable way. The common categories of weakening would be 1. making the evidence less relevant or trustworthy 2. weakening the connection between the evidence and the conclusion 3. making the conclusion seem less plausible 4. suggesting an alternate explanation or framing for the evidence Only about 1-2% of correct answers weaken by directly attacking the credibility of the evidence. Only about 10-15% of correct answers weaken by just attacking the plausibility of the conclusion. The vast majority respond to the evidence, showing that the evidence isn't relevant to the conclusion, is compatible with the conclusion being false, or can be explained in some way other than how the conclusion is explaining it.
@LSATLab
@LSATLab Год назад
Since the conclusion said "airlines are overconcerned about business travelers, they should instead focus on leisure travelers", then we as the defense attorney would be arguing, "Wrong -- it's correct how much they're focusing on business travelers, relative to leisure travelers".
@mairobertson9349
@mairobertson9349 Год назад
So out of the four categories you just mentioned which is the best method to use? Or all of them get used at one point
@LSATLab
@LSATLab Год назад
@@mairobertson9349 Yup, they all get used at some point, but as I said before a couple of those categories only account for around 15% of answers total. The most generic advice for Weaken is to just head to the answers thinking, "GIVEN THAT [evidence], HOW CAN I ARGUE [anti-conclusion]" So if the argument were, "Bob never gives his wife flowers on their anniversary. Thus, he is a bad husband." You'd go to the answers thinking, "GIVEN THAT bob doesn't give his wife flowers on their anniversary, HOW CAN I STILL ARGUE that he's a *good* husband?" Or alternatively, you can think about how to respond to the evidence by framing it this way, "GIVEN THAT I'M ARGUING that bob is a good husband, HOW CAN I EXPLAIN THAT he doesn't give his wife flowers on their anniversary?" In general answers either have to respond to the evidence or outweigh the evidence. We can either try to make it seem like it's not a bad thing that he doesn't give his wife flowers, or we can accept that it's a bad thing but outweigh it with other considerations that make it seem more like he's a good husband.
@mairobertson9349
@mairobertson9349 Год назад
Thank you!
@ilsaann4988
@ilsaann4988 2 года назад
How are 78% of weakening questions comparison and 52% of them are causation? Could someone please explain how this math works
@LSATLab
@LSATLab 2 года назад
Hi Ilsa, overlapping sets is how the math adds up. Arguments can utilize more than one type of reasoning structure.
@ilsaann4988
@ilsaann4988 2 года назад
@@LSATLab Oh okay that makes sense. Thank you for the videos!
@mohammadnaghshineh9311
@mohammadnaghshineh9311 4 года назад
Hi Thanks for your nice videos The pictures and pdf files are too small Would you please introduce a suitable website that this files are available
@LSATLab
@LSATLab 4 года назад
Hey Mohammad, not sure what exactly you mean by pdf files. Can you elaborate? Are you saying that the text in the videos is too small?
@mohammadnaghshineh9311
@mohammadnaghshineh9311 4 года назад
LSAT Lab yes The text is too small Thank you for your kind
@LSATLab
@LSATLab 4 года назад
@@mohammadnaghshineh9311 One more follow up. Are you referencing the text of the actual LSAT questions specifically? Or is it a general issue that affects other segments of text as well? Thank you so much for the feedback, I'd like to make this video series as helpful as possible for as many people as possible.
@mohammadnaghshineh9311
@mohammadnaghshineh9311 4 года назад
LSAT Lab I am interested in mathematical Logic I enjoy these videos Thank you
@martini87c
@martini87c 2 года назад
I enjoy these LSAT explanation videos, but the consumer advocate question at 18:06 is a really bad example
@LSATLab
@LSATLab 2 года назад
Thanks. What do you mean by "a bad example", out of curiosity? Too hard / too easy? Not representative of this question type?
@martini87c
@martini87c 2 года назад
@@LSATLab I would say not representative of this question especially for folks that are identifying these types of questions as their weak spots on the lsat such as myself …. Maybe easier question would have been better to kind of help link the analysis points /break downs first and then this question could have been a follow up ? Otherwise I do enjoy your channel and information for lsat thank you ☺️
@LSATLab
@LSATLab 2 года назад
@@martini87c , thanks for the feedback!
@riyakapoor3218
@riyakapoor3218 4 года назад
Is weakening argument always against the conclusions??
@samdavisok
@samdavisok 4 года назад
It's against the SUPPORT relationship between the conclusion and premises/evidence.
@samdavisok
@samdavisok 4 года назад
What I've gathered is, it's important to look at it like because of the causation type weakening questions
Далее
Strengthen | LSAT Logical Reasoning
24:28
Просмотров 28 тыс.
Famous Flaws | LSAT Logical Reasoning
24:50
Просмотров 59 тыс.
Flaw | LSAT Logical Reasoning
22:23
Просмотров 59 тыс.
Lesson 3: LSAT Logical Reasoning (Part 1)
1:14:16
Просмотров 15 тыс.
170+ LR: Are you doing LSAT Weaken Questions wrong?
9:23
10 Things You NEED To Do For The LSAT
12:19
Просмотров 2,7 тыс.
Most Supported | LSAT Logical Reasoning
25:42
Просмотров 22 тыс.
Necessary Assumption LSAT Logical Reasoning
26:41
Просмотров 13 тыс.
Sufficient Assumption | LSAT Logical Reasoning
22:38
Просмотров 52 тыс.